exp(x)のMaclaurin級数の一般化

関数項級数

$$ \sum_{n=0}^{\infty}\frac{x^n}{n!} $$

はとても有名で、 \( e^{x} \) のMaclaurin展開として知られています。では、これを一般化した級数

$$ \sum_{n=0}^{\infty}\frac{x^n}{n!_k} $$

はどのような関数になるでしょうか (ただし \( k \) は正の整数)。ここで、\( n!_k \) は多重階乗で、詳しくは

twelve-sakuya.hatenablog.com

をご覧ください。

今回は実数の範囲で考えるとして、関数 \( f_k\colon\mathbb{R}\to\mathbb{R} \) を

$$ f_k(x):=\sum_{n=0}^{\infty}\frac{x^n}{n!_k} $$

とおきます。ここで、本当に定義域を \( \mathbb{R} \) としてよいかは分からないので、まず \( f_k \) の収束半径を求めてみましょう。d'Alembertの収束判定法から

$$ \lim_{n\to\infty}\left|\frac{1/(n+1)!_k}{1/n!_k}\right|=\lim_{n\to\infty}\frac{n!_k}{(n+1)!_k} $$

を考えます。多重階乗は \(!\) の数で割った余りによって漸近挙動が変わるので、\( n=kp-q \) (\( 1\le q\le k -1 \))とおくと、\( n\to\infty\) のとき \( p\to\infty \) であり、上の記事から

$$ n!_k=(kp-q)!_k\sim\frac{\sqrt{2\pi}}{\Gamma(1-\frac{q}{k})}p^{\frac{1}{2}-\frac{q}{k}}\left(\frac{kp}{e}\right)^p \\\\ (n+1)!_k=(kp-q+1)!_k\sim\frac{\sqrt{2\pi}}{\Gamma(1-\frac{q-1}{k})}p^{\frac{1}{2}-\frac{q-1}{k}}\left(\frac{kp}{e}\right)^p $$

なので

$$ \frac{n!_k}{(n+1)!_k}\sim\frac{\Gamma(1-\frac{q-1}{k})}{\Gamma(1-\frac{q}{k})}p^{-\frac{1}{k}}\to0\quad(p\to\infty) $$

より

$$ \lim_{n\to\infty}\left|\frac{1/(n+1)!_k}{1/n!_k}\right|=0 $$

がわかります ( \( q=0 \) のときは \( k \) ずらせばよいです)。ゆえに、収束半径は \( \infty \)となり、すなわち任意の \( x\in\mathbb{R} \) で \( f_k(x) \) は収束します。

さて、\( f_k \) を何らかの関数で表すことを考えるわけですが、母関数を求める常套手段として、微分方程式を作るというのがあります。また、多重階乗は帰納的に定義されていたことから、\( f_k \) を微分すれば同じ形を作りだせそうです。よって、まずは \( f_k \) を微分することを思いつきます。ここで、級数微分が出てきますが、冪級数は収束半径内なら項別微分が可能なことが知られているので、気にせず微分できて

$$ \begin{align*} {f_k}'(x)=\sum_{n=1}^{\infty}\frac{n}{n!_k}x^{n-1}&=\sum_{n=1}^{k -1}\frac{n}{n!_k}x^{n-1}+\sum_{n=k}^{\infty}\frac{n}{n!_k}x^{n-1} \\\\ &=\sum_{n=1}^{k -1}\frac{n}{n}x^{n-1}+\sum_{n=k}^{\infty}\frac{1}{(n-k)!_k}x^{n-1} \\\\ &=\sum_{n=1}^{k -1}x^{n-1}+\sum_{n=0}^{\infty}\frac{1}{n!_k}x^{n+k -1} \\\\ &=\sum_{n=1}^{k -1}x^{n-1}+x^{k -1}f_k(x) \\\\ \end{align*} $$

となります。これにて、微分方程式

$$ {f_k}'(x)-x^{k -1}f_k(x)=\sum_{j=1}^{k -1}x^{j-1} $$

が得られました (「\( n= \)」 のままだと個人的にちょっと気持ち悪いので \( n \) を \( j \) に変えました)。ここで、両辺に \( e^{-\frac{x^{k}}{k}} \) をかけると、左辺は

$$ e^{-\frac{x^k}{k}}({f_k}'(x)-x^{k -1}f_k(x))=\left(e^{-\frac{x^k}{k}}f_k(x)\right)' $$

とできるので

$$ \left(e^{-\frac{x^k}{k}}f_k(x)\right)'=e^{-\frac{x^k}{k}}\sum_{j=1}^{k -1}x^{j-1} $$

となります。さらに

$$ e^{-\frac{0^k}{k}}f_k(0)=1 $$

なので、微分積分学の基本定理より

$$ e^{-\frac{x^k}{k}}f_k(x)=\int_{0}^{x}e^{-\frac{t^k}{k}}\sum_{j=1}^{k -1}t^{j-1}dt+1 $$

を得ます。この右辺について考えてゆきましょう。総和の部分を等比数列の和の公式を用いて綺麗にしたい気持ちはありますが、とりあえずはそのままで考えてみます。すると

$$ \begin{align*} \int_{0}^{x}e^{-\frac{t^k}{k}}\sum_{j=1}^{k -1}t^{j-1}dt+1&=\sum_{j=1}^{k -1}\int_{0}^{x}t^{j-1}e^{-\frac{t^k}{k}}dt+1 \\\\ &=\sum_{j=1}^{k -1}\int_{0}^{\frac{x^k}{k}}\left(k^{\frac{1}{k}}t^{\frac{1}{k}}\right)^{j-1}e^{-t}\frac{k^{\frac{1}{k}}}{k}t^{\frac{1}{k}-1}dt+1\quad\left(\because t\mapsto k^{\frac{1}{k}}t^{\frac{1}{k}}\right) \\\\ &=\sum_{j=1}^{k -1}k^{\frac{j}{k}-1}\int_{0}^{\frac{x^k}{k}}t^{\frac{j}{k}-1}e^{-t}dt+1 \\\\ &=\sum_{j=1}^{k -1}k^{\frac{j}{k}-1}\gamma\left(\frac{j}{k},\frac{x^k}{k}\right)+1 \end{align*} $$

となります。ここで、\( \gamma(\cdot,\cdot)\colon[0,\infty)\times[0,\infty)\to\mathbb{R} \) は第一種不完全gamma関数で

$$ \gamma(a,x):=\int_{0}^{x}t^{a-1}e^{-t}dt $$

で与えられます。定義から分かるように

$$ \lim_{x\to\infty}\gamma(a,x)=\Gamma(a) $$

となります (これが「不完全」な所以)。これよりさらに

$$ \begin{align*} \sum_{j=1}^{k -1}k^{\frac{j}{k}-1}\gamma\left(\frac{j}{k},\frac{x^k}{k}\right)+1&=\sum_{j=1}^{k}k^{\frac{j}{k}-1}\gamma\left(\frac{j}{k},\frac{x^k}{k}\right)-\gamma\left(1,\frac{x^k}{k}\right)+1 \\\\ &=\sum_{j=1}^{k}k^{\frac{j}{k}-1}\gamma\left(\frac{j}{k},\frac{x^k}{k}\right)-\int_{0}^{\frac{x^k}{k}}e^{-t}dt+1 \\\\ &=\sum_{j=1}^{k}k^{\frac{j}{k}-1}\gamma\left(\frac{j}{k},\frac{x^k}{k}\right)+e^{-\frac{x^k}{k}} \end{align*} $$

とでき、ゆえに

$$ f_k(x)=e^{\frac{x^k}{k}}\sum_{j=1}^{k}k^{\frac{j}{k}-1}\gamma\left(\frac{j}{k},\frac{x^k}{k}\right)+1 $$

が従います。試しに \( k=1 \) としてみると

$$ f_1(x)=e^{x}\sum_{j=1}^{1}1^{j-1}\gamma\left(j,x\right)+1=e^x\int_{0}^{x}e^{-t}dt+1=e^x $$

となり、級数において \( k=1 \) としたものと一致します。

以上より題意の級数

$$ \sum_{n=0}^{\infty}\frac{x^n}{n!_k}=e^{\frac{x^k}{k}}\sum_{j=1}^{k}k^{\frac{j}{k}-1}\gamma\left(\frac{j}{k},\frac{x^k}{k}\right)+1 $$

となることが分かりました。「\( +1 \)」 が目障りなら

$$ \sum_{n=1}^{\infty}\frac{x^n}{n!_k}=e^{\frac{x^k}{k}}\sum_{j=1}^{k}k^{\frac{j}{k}-1}\gamma\left(\frac{j}{k},\frac{x^k}{k}\right) $$

としてやると綺麗ですね。

ちなみに、今回求めたことを用いれば

$$ \lim_{x\to\infty}e^{-\frac{x^k}{k}}\sum_{n=0}^{\infty}\frac{x^n}{n!_k}=\lim_{x\to\infty}\left(\sum_{j=1}^{k}k^{\frac{j}{k}-1}\gamma\left(\frac{j}{k},\frac{x^k}{k}\right)+e^{-\frac{x^k}{k}}\right)=\sum_{j=1}^{k}k^{\frac{j}{k}-1}\Gamma\left(\frac{j}{k}\right) $$

といった極限を求めることもできます。